2
$\begingroup$

Assume that $ \int_{1}^{+\infty} f(x) dx $ converges (as improper Riemann integral). Does there necessarily exist some $ \xi \in (1, +\infty) $ such that $$ \int_{1}^{+\infty} \frac{f(x)}{x} dx = \int_{1}^{\xi} f(x) dx ? $$

My Attempt :

If $f(x)$ is a positive function, then we can apply the second mean value theorem for integrals on the interval $[1, n]$. This guarantees the existence of $\xi_n \in (1, n)$ such that $$ \int_{1}^{n} \frac{f(x)}{x} dx = \int_{1}^{\xi_n} f(x) dx. $$ Furthermore, the sequence $\{\xi_n\}$ must be bounded. Otherwise, if it were unbounded, there would exist a subsequence $\xi_{n_k} \to +\infty$ such that $$ \int_{1}^{\infty} f(x) dx = \int_{1}^{\infty} \frac{f(x)}{x} dx, $$ which leads to a contradiction since $f(x) > 0$ implies $\frac{f(x)}{x} < f(x)$ for $x > 1$, making the equality impossible.

Difficulty: However, the problem does not actually require $f(x)$ to be a positive function, which is where I encounter difficulties.

$\endgroup$
1
  • $\begingroup$ Riemann , and $f$ is not assumed to be continuous, I don't know whether integration by parts is valid without continuity. $\endgroup$ Commented Oct 24 at 16:11

1 Answer 1

1
$\begingroup$

The function $F(x) = \int_1^x f(t) \, dt$ is continuous on $[1, +\infty)$ and has a limit for $x \to +\infty$, therefore $$ m = \inf \{ F(x) \mid x \ge 1 \} \, ,\\ M = \sup \{ F(x) \mid x \ge 1 \} \, . $$ both exist as (finite) real numbers.

Integration by parts (see for example Integration by parts for Riemann integrable fucntion) gives $$ \int_1^b \frac{f(x)}{x} \,dx = \frac{F(b)}{b} + \int_1^b \frac{F(x)}{x^2} \,dx $$ for any $b > 1$ and we can take the limit $b \to +\infty$: $$ \int_1^{+\infty} \frac{f(x)}{x} \,dx = \int_1^{+\infty} \frac{F(x)}{x^2} \,dx \, . $$ We have $$ m = \int_1^{+\infty} \frac{m}{x^2} \,dx \le \int_1^{+\infty} \frac{F(x)}{x^2} \,dx \le \int_1^{+\infty} \frac{M}{x^2} \,dx = M \, . $$ Equality on either side can only hold if $F$ is constant. In that case is $f$ identically zero and the desired identity holds trivially for any $\xi > 1$.

So we can assume that $$ m < \int_1^{+\infty} \frac{f(x)}{x} \,dx < M $$ and conclude that $$ \int_1^{+\infty} \frac{f(x)}{x} \,dx = F(\xi) $$ for some $\xi > 1$.

$\endgroup$
1
  • 1
    $\begingroup$ Thank you! The proof that $\xi > 1$ is ingenious, and I've learned a great deal from it. $\endgroup$ Commented Oct 24 at 16:54

You must log in to answer this question.

Start asking to get answers

Find the answer to your question by asking.

Ask question

Explore related questions

See similar questions with these tags.